Math, asked by mansijoshi6646, 10 months ago

Express the following in p by q form
1.72 4bar

Answers

Answered by parvati050180
0

Answer:

Step-by-step explanation :Hope it helps you

Please mark it as brainliest pls

Attachments:
Similar questions